LSAT and Law School Admissions Forum

Get expert LSAT preparation and law school admissions advice from PowerScore Test Preparation.

 jostomel
  • Posts: 12
  • Joined: Jan 09, 2015
|
#17967
Can you explain why D is not correct. I understand how the book explanation got to A, Family cars on 1, but don't the inferences show that Sports cars must also be on floor 3 (e)? The book doesn't do a very good job describing this problem, maybe you can shed some light on it.
 Emily Haney-Caron
PowerScore Staff
  • PowerScore Staff
  • Posts: 577
  • Joined: Jan 12, 2012
|
#17972
Hi jostomel,

Thanks for the question! The key here is that there do not HAVE to be sports cars; it is possible that each floor has family cars, including the third floor. Does that make sense?
 jostomel
  • Posts: 12
  • Joined: Jan 09, 2015
|
#17975
Yes that makes sense, I was assuming that both sports and family cars had to be present in the game. thanks

Get the most out of your LSAT Prep Plus subscription.

Analyze and track your performance with our Testing and Analytics Package.